51
LAWYERS WITHOUT BORDERS? Keeping the UPL Out of the MJP Prepared by William B. Dunn for The American College of Real Estate Lawyers March 2015 “Once upon a time, lawyers worked at a desk. The desk was in a place (a state) and the government of that state (via the courts granting a license) validated the lawyer's presence at that desk in that state. Without that validation there was no lawyer and no desk. The license stopped at the border of the state. . . . In a sense, each jurisdiction was a cocoon.” 1 I. Where Matters, But Why? A. Importance of Geography. The effect of geographical validation is that a lawyer stops being a licensed lawyer when crossing jurisdictional boundaries. Lawyers engaged in transactional practice today will often find themselves beyond their borders. Consider these common situations: Representing a client in a jurisdiction in which lawyer is licensed to practice law in negotiating and documenting a transaction involving property in a jurisdiction in which lawyer is not licensed to practice law. (beyond border law) Representing a client in a jurisdiction in which lawyer is not licensed to practice, in a transaction involving 1 STEPHEN GILLERS, A Profession If You Can Keep It: How Information Technology and Fading Borders Are Reshaping the Law Marketplace and What We Should Do About It , 63 HASTINGS L. J. 953, 961 (2012). [Author’s note : The article cited, authored by a member of the ABA Commission on Multijurisdictional Practice, is an outstanding examination of cross border law practice and is a significant source for this paper.] 1 200762123.7 09999/09998-0108

cdn.ymaws.com  · Web viewrelated to matters such as education, demonstration of competence such as success in a bar examination, and character. Geographical validation is based

  • Upload
    tranbao

  • View
    215

  • Download
    0

Embed Size (px)

Citation preview

LAWYERS WITHOUT BORDERS?Keeping the UPL Out of the MJP

Prepared by William B. Dunn

for The American College of Real Estate Lawyers

March 2015

“Once upon a time, lawyers worked at a desk. The desk was in a place (a state) and the government of that state (via the courts granting a license) validated the lawyer's presence at that desk in that state. Without that validation there was no lawyer and no desk. The license stopped at the border of the state. . . . In a sense, each jurisdiction was a cocoon.”1

I. Where Matters, But Why?

A. Importance of Geography. The effect of geographical validation is that a lawyer stops being a licensed lawyer when crossing jurisdictional boundaries. Lawyers engaged in transactional practice today will often find themselves beyond their borders. Consider these common situations:

Representing a client in a jurisdiction in which lawyer is licensed to practice law in negotiating and documenting a transaction involving property in a jurisdiction in which lawyer is not licensed to practice law. (beyond border law)

Representing a client in a jurisdiction in which lawyer is not licensed to practice, in a transaction involving property in a jurisdiction in which lawyer is not licensed to practice law. (beyond border client)

Representing a client in a jurisdiction in which lawyer is licensed to practice law in a transaction involving property in a jurisdiction in which lawyer is licensed to practice law while lawyer is physically located in a jurisdiction in which lawyer is not licensed to practice law. (beyond border location)

Where is the lawyer “practicing law?”

The Restatement of the Law, Third, The Law Governing Lawyers (referred to in this paper as “RESTATEMENT”)2, in §2 recites the premise that in order to become a lawyer and qualify to practice law in a jurisdiction the prospective lawyer must comply with its local requirements

1 STEPHEN GILLERS, A Profession If You Can Keep It: How Information Technology and Fading Borders Are Reshaping the Law Marketplace and What We Should Do About It, 63 HASTINGS L. J. 953, 961 (2012). [Author’s note: The article cited, authored by a member of the ABA Commission on Multijurisdictional Practice, is an outstanding examination of cross border law practice and is a significant source for this paper.] 2 The American Law Institute, 2000.

1

200762123.7 09999/09998-0108

related to matters such as education, demonstration of competence such as success in a bar examination, and character.

Geographical validation is based on the fundamental principle that each state administers a separate system for admission to practice law through its judiciary. The right of, and preference for, each state to do so was affirmed as the first tenet of the Report of the ABA Commission on Multijurisdictional Practice in 2002.3 This principle is realized through

each state’s establishment of requirements for education of those seeking admission to the bar of that state;

each state’s examination and approval of the character of each candidate for admission; and

each state’s determination that lawyers admitted there possess competence to practice, measured by passing an examination on the law of the admitting jurisdiction.

B. Admission to Practice Law. A person may be admitted to practice law generally by complying with the requirements of the jurisdiction for admission. A lawyer admitted elsewhere may be admitted on a matter-by-matter basis before a tribunal or administrative agency through pro hac vice arrangements. These are discretionary with a court, and apply only to matters in a proceeding. A number of states allow admission by motion to gain permanent admission without the need to take the full bar examination.4 Where adopted, a rule for admission by motion has often been conditioned on reciprocity – comity of the applicant lawyer’s admitting jurisdiction for motion admissions of lawyers admitted in the on-motion state.5

C. Regulation of the Profession. The legal profession is regulated in each jurisdiction principally by the judicial branch of government, but that regulation is often supplemented by the legislative branch, and also by the administrative branch governing agencies.6 The rules of

3 American Bar Association, REPORT OF THE COMMISSION ON MULTIJURISDICTIONAL PRACTICE (August 2002) [cited here as the MJP Report], accessed at http://www.americanbar.org/content/dam/aba/migrated/cpr/mjp/pdf. Additional information about the Commission and adaptation of its recommendations through rules of professional conduct based on Model Rule 5.5(b), (c), and (d) can be accessed at http://www.americanbar.org/groups/professional_responsibility/committees_commissions/commission_on_multijurisditional_practice.html. 4 The America Bar Association has promulgated a Model Rule on Admission by Motion, last amended in 2012. http://www.americanbar.org/content/dam/aba/administrative/professional_responsibility/model_rule_admission_motion.authcheckdam.pdf. This Model Rule discourages requirements not within the text of the rule, including reciprocity.5 As of April 2014, 42 states provided for admission by motion or other process. About half required reciprocity – the lawyer’s home jurisdiction must extend the same accommodation to lawyers from the temporarily admitting jurisdiction. http://www.americanbar.org/content/dam/aba/administrative/professional_responsibility/admission_motion_rules.authcheckdam.pdf. As a result of a complaint filed by the National Association for the Advancement of Multijurisdictional Practice v. Richard W. Roberts, Chief Judge, et al (US DC Civil No. 1:13-CV-1963 TFH), the District Court in 2014 revised DC court rule LCvR 83.8(a) to eliminate the reciprocity requirement in the District of Columbia federal courts. Similar actions of the NAAMJP have not been as successful, most recently in the 9th Circuit Court of Appeals as regards reciprocity requirements of Arizona (Nat’l Ass’n for the Advancement of Multijurisdictional Practice v. Berch, 973 F. Supp. 2d 1082) and in the Eastern District of Pennsylvania as regards the Pennsylvania requirement (Nat’l Ass’n for the Advancement of Multijurisdictional Practice et al. v. Castille, et al, 2014 U.S. Dist. LEXIS 171341. 6 For a discussion of the system of lawyer regulation, see STEPHEN GILLERS, REGULATION OF LAWYERS: PROBLEMS OF LAW AND ETHICS 8-10 (9th ed. 2012).

2

200762123.7 09999/09998-0108

regulation of the legal profession are found in statutes, court rules, rules of professional conduct, and rules of lawyer discipline adopted in each jurisdiction. It is fair to observe that the independence of each jurisdiction’s judiciary in establishing and enforcing such regulation is a zealously defended right, without conscious effort to establish uniformity in regulation among jurisdictions, and with begrudging comity.

The broadly advanced purposes of lawyer regulation are:

Protection of the public, including clients. Protection and advancement of the integrity of the profession in terms of its core values,

reflected in ethical obligations and professional responsibility. Establishment of criteria and qualifications for persons purporting to practice law or

related endeavor (ostensibly serving the first two purposes).

D. Unauthorized Practice of Law (“UPL”). Central to each state’s regulation of law practice consistent with the principles of geographical limitation are rules governing unauthorized practice of law (“UPL”). These rules, which prohibit equally lawyers not admitted or otherwise authorized to practice in the jurisdiction and nonlawyers from engaging in the practice of law in the jurisdiction, are in force in every state. The effect of these rules is that lawyers may practice law only in those jurisdictions where admitted or otherwise authorized to do so.

The Model Rules of Professional Conduct,7 which is the basis of Rules of Professional Conduct (“RPC”) in all states (except California) and in the District of Columbia,8 provide a UPL rule, the violation of which may lead to disciplinary action. Model Rule 5.5(a),9 itself labeled “Unauthorized Practice of Law,” states:

A lawyer shall not practice law in a jurisdiction in violation of the regulation of the legal profession in that jurisdiction, or assist another in doing so.

This Model Rule reads that a lawyer (subject to this Model Rule in this jurisdiction, here) shall not practice law where doing so violates regulation of the legal profession in “that” jurisdiction (there). Its predecessor rule, DR 3-101(B) of the Model Code of Professional Responsibility and the first version of Model Rule 5.5 adopted in 1983 were even more clearly beyond-the-border focused, using the phrase “… in a jurisdiction where doing so violates …”;10 and the rule as

7 All Model Rules of Professional Conduct quoted in this paper are the ABA Model Rules 2014 Edition, amended through August 2013, ©American Bar Association (referred to herein as the Model Rules). Reprinted with permission.8 For information about state adoption of Rules of Professional Conduct based on the Model Rules, visit the website of the ABA Center for Professional Responsibility at: http://www.americanbar.org/groups/professional_responsibility/publications/model_rules_of_professional_conduct.html. The ABA does not post a catalog of variations among the states in adopting rules of professional conduct based on the Model Rules, but does provide a link to all state rules of professional conduct, accessed at: http://www.americanbar.org/groups/professional_responsibility/resources/links_of_interest.html. 9 The full text of MRPC 5.5 and Comment to it is set out in Appendix A. 10 Legislative history of the Model Rules does not indicate any intent to change the aim of Model Rule 5.5(a) in the evolution of its wording. See A LEGISLATIVE HISTORY: THE DEVELOPMENT OF ABA MODEL RULES OF PROFESSIONAL CONDUCT, 1982-2005, American Bar Assoc. 2006 (referred to herein as A LEGISLATIVE HISTORY),

3

200762123.7 09999/09998-0108

presently stated continues to carry out that purpose. Unless this Model Rule statement is to be read as if a biblical pronouncement of general principle, this is a rule of comity. The Model Rule also prohibits assisting another person11 to do what this Model Rule prohibits.

The geographical heritage of lawyer regulation carries into this Model Rule. The Model Rule refers to practice of law “in” a jurisdiction, continuing the emphasis on where the lawyer is when the lawyer practices law. This raises some obvious questions:

If the lawyer was in the lawyer’s state of admission when practicing the law of another jurisdiction in violation of its regulation, would the Model Rule be violated? (Many would think not -- See, e.g., Comment e to Restatement §3 – but the regularity, frequency, and degree of that practice could translate to sufficient presence.)

Does the lawyer really have to be there? Why not “of” a jurisdiction? o Is a New York (only) lawyer working in New York forming a Delaware

corporation for a New York client practicing law “in” Delaware? If the lawyer working in New York advises the client about Delaware law, is the lawyer practicing law “in” Delaware? If the lawyer is sitting in a hotel in Wilmington, can the New York lawyer do the same things?

o Is a Michigan (only) lawyer using her home office in Colorado, advising clients in Michigan, Illinois, and California practicing law “in” Colorado? In Illinois? In California? In Michigan?

o If the aim of the Model Rule is protection of the public against rendition of legal services by unqualified persons (persons not members of the bar12), should Colorado care? Should New York or Michigan or Delaware care? Would it matter if, for example, Delaware did not care?

Does it matter how the lawyer is there? Is physical presence needed? Can virtual presence be enough?

Should it matter for what purpose the lawyer is there? Should regulators look to in whose interest is it that a certain public be protected, or should the state of licensure protect that interest wherever it exists?

It is reasonable to conclude that physical presence in a jurisdiction where a lawyer is not admitted to practice law may be more obviously risky but that “presence” may be found in other activities.

Underlying this Model Rule are two matters of fundamental inquiry: what is the practice of law, and what is the unauthorized practice of law.

1. What is the practice of law? The answer to this question seems fundamentally essential to application of a rule prohibiting unauthorized practice of law, but there is no

at 613-630.11 Although the syntax of the Model Rule would indicate that “another” is a lawyer, the original language referred to persons who were not members of the bar. Consistent with the principle that a lawyer's license ends at the state line, the clarifying reference of the original rule is not necessary or useful for lawyers; however, the Model Rule as drafted appears to omit non-lawyers. That is not its intent. The word “another” is intended to mean anybody. See, Discussion accompanying Report 201(B) of the Commission on Multijurisdictional Practice proposing amendment to Model Rule 5.5, 2002, in A LEGISLATIVE HISTORY, supra note 10, at 625.12 See Comment [2] to Model Rule 5.5.

4

200762123.7 09999/09998-0108

common definition of the term. An ABA Task Force on the Model Definition of the Practice of Law was appointed in 2002 to create a model definition. The work of this Task Force was motivated by situations where nonlawyers were providing services that were difficult to categorize under existing law as being, or not being, the delivery of legal services. The Task Force’s draft of a model definition drew many comments and objections, and the eventual report of the Task Force concluded that the subject was a matter for each state and territory to define, but that a basic premise to be observed was that the practice of law is the application of legal principles and judgment to the circumstances or objectives of another person or entity.”13

As discussed below, Model Rules 5.5(c) and (d) use the term “provide legal services” in permitting certain conduct that might otherwise violate Model Rule 5.5(b), the “in this jurisdiction” counterpart of Model Rule 5.5(a), which also uses the term “practice of law” and “practice law.” Whether providing legal services is the same as practice of law is unclear at best. Comment [13] to Model Rule 5.5 provides a broad statement that in understanding Model Rule 5.5(c)(4), legal services includes services that a nonlawyers may perform but are considered the practice of law when performed by a lawyer. This statement appears to support a definition of the practice of law based on who is doing the work, not the nature of the work itself.

State efforts to define the term are often tautological or vague.14 Some examples are:

Michigan – Dressel v. Ameribank, 468 Mich 557, 664 NW2d 151(2002): “… [A] person engages in the practice of law when he (sic) counsels or assists another in matters that require the use of legal discretion and profound legal knowledge.” Dressel, 468 Mich at 569.

Texas – Texas Government Code Section 81.101(a):“…[T]he ‘practice of law’ means the preparation of a pleading or other document incident to an action or proceeding on behalf of a client before a judge in court as well as a service rendered out of court, including the giving of advice or the rendering of any service requiring the use of legal skill or knowledge, such as preparing a will, contract, or other instrument, the legal effect of which under the facts and conclusions involved must be carefully determined.”

Washington – Washington Court Rules General Rule 24(a):“The practice of law is the application of legal principles and judgment with regard to the circumstances or objectives of another entity or person(s) which requires the knowledge and skill of a person trained in the law. This includes but is not limited to: [here follows 4 specific examples].”

Arizona – Arizona Supreme Court Rule 31:“‘Practice of law’ means providing legal advice or services to or for another by:

(1) preparing any document in any medium intended to affect or secure legal rights for a specific person or entity;(2) preparing or expressing legal opinions;

13 American Bar Association, REPORT OF THE TASK FORCE ON THE MODEL DEFINITION OF THE PRACTICE OF LAW (August 2003) http://www.americanbar.org/groups/professional_responsibility/task_force_model_definition_practice_law.html. 14 The most recent compilation of information about state definitions of practice of law and of unauthorized practice of law maintained by the American Bar Association can be found at http://www.americanbar.org/groups/professional_responsibility/resources/client_protection/client.html#UPL.

5

200762123.7 09999/09998-0108

(3) representing another in a judicial, quasi-judicial, or administrative proceeding, or other formal dispute resolution process such as arbitration and mediation;(4) preparing any document through any medium for filing in any court, administrative agency or tribunal for a specific person or entity; or(5) negotiating legal rights or responsibilities for a specific person or entity.”

Some jurisdictions provide exceptions and exclusions from the regulated penumbra of the practice of law. Examples:

Washington – Washington Court Rules General Rule 24(b): “Whether or not they constitute the practice of law, the following are permitted: [then follows 10 specific and one non-specific (other activities permitted by the Supreme Court as not constituting unauthorized practice or permitted under a regulatory system established by the Supreme Court) activities].”

Texas – Texas Government Code Section 81.101(c) provides that practice of law does not include the publication, distribution, or sale of written materials, software, and similar products through any medium as long as the products say that they are not a substitute for the advice of an attorney.

2. What is the “unauthorized” 15 practice of law ? Unauthorized practice of law is typically defined as the practice of law by a person not authorized to do so under the rules of the jurisdiction. The definition is particularly difficult when a jurisdiction has not artfully defined “practice of law” or what it means to “practice law.”

By implication, Model Rule 5.5(a) appears to provide yet another term by which to understand what UPL is: practicing law in violation of the regulation of the legal profession. This would include not only practicing where not admitted or otherwise authorized to practice law, but practicing in a manner that violates any lawyer regulation rule, even where a lawyer is also admitted to practice. That term, which broadly encompasses any law or rule regulating the profession, was intended to refer only to regulation governing the unauthorized practice of law,16 a view supported by Comment to the original Model Rule adopted in 198317 and by the caption

15 In some jurisdictions, e.g. Washington, this is referred to as “unlawful” practice of law. The ABA Standing Committee on Client Protection uses the term “unlicensed” in its publications (see e.g., note 14 supra and note 19 infra). Some difference may be perceived between “unauthorized” and “unlicensed” – the former might refer to a lawyer licensed but not admitted in the jurisdiction and the latter might mean a person not licensed to practice law anywhere; but no such a formal distinction has been yet recognized. 16 Originally proposed as a part of Model Rule 8.4 “Professional Misconduct” when the Model Rules were first recommended for adoption in 1982, the text now appearing substantively as Model Rule 5.5(a) was made a separate Model Rule at the recommendation of the Standing Committee on Unauthorized Practice of Law. See, A LEGISLATIVE HISTORY: THE DEVELOPMENT OF ABA MODEL RULES OF PROFESSIONAL CONDUCT, 1982-2005, American Bar Assoc. 2006, at 614. The use of the term “regulation of the legal profession” if intended to mean a specific part of that regulation, unauthorized practice rules, does not reflect well on legal drafting practices. Read as written, the Model Rule is a broader but inclusive version of Model Rule 8.4(a), and has no specific relationship to unauthorized practice of law. 17 The rationale of this Model Rule was stated in Comment to the Model Rule when first adopted as: “Limiting the practice of law to members of the bar protects the public from rendition of legal services by unqualified persons.” The rationale would seem to apply only to the original reference to non-lawyers, or perhaps to lawyers not members of this jurisdiction's bar. The broad reference to “regulation” in the Model Rule and the focus of Comment, local bar membership, did not jibe. The statement of underlying rationale is retained for the present Model Rule in Comment

6

200762123.7 09999/09998-0108

to the Model Rule. The broader meaning of this rule is expressed in Model Rule 8.4(a) and addressed also in Model Rule 8.5.

Regulation of unauthorized practice of law may be the subject of statute or a court rule, or both. Legislation typically defers to the power of the courts of the jurisdiction to speak to the subject beyond the legislation. The conduct encompassed in some definitions may be broader than practice of law. For example, a Michigan statute18 provides that “[a] person shall not practice law or engage in the law business … unless the person is regularly licensed and authorized to practice law in this state.” (emphasis added) Under the Michigan law, the term “law business” is not defined. Some jurisdictions recognize that certain activities do not constitute the regulated practice of law, narrowing to some degree that which is unauthorized. These redefinitions look at the activity itself, not the means of the activity (how and where it is done) or the effect of the activity (for whom or on what).

Enforcement of UPL may be left to the state bar (e.g. Michigan), an attorney general (e.g. Washington), or another entity or agency.19

E. The Boundary Premise in Question. The historic view that a lawyer is a lawyer only within the boundaries of the admitting jurisdiction has been questioned.20 The reasons for breaking boundaries include:

There are increasingly common standards for bar admission. The ability of a jurisdiction to administer the bar and the practice of law does not need to depend on the status of admission to the state’s bar. Admission to practice anywhere ought to be reasonably indicative of admissibility elsewhere.

Law schools are accredited under common standards. Bar admission educational prerequisites are relatively standard, and most require graduation from an accredited law school.

Character and fitness inquires have been constrained by law, and often depend on a national clearinghouse of information. Social media may be more revealing than formalities imposed by a state bar. The continuing character and fitness of a practicing lawyer is the subject of administration somewhere.

Bar examinations are more standardized. Specific legal knowledge of the law of a jurisdiction can be easily accessed through electronic means, among many resources. In addition, law has become increasingly uniform through Uniform Laws and the Restatements of the Law.

Historic admission requirements have given way through pro hac vice admission on a matter-by-matter basis and through admission by motion rules in many jurisdictions.

Lawyers and clients are physically and virtually mobile. Practice is less office based than client based, and going wherever the client is to be served is to be expected.

[2]. The RPC of a jurisdiction apply to any lawyer practicing law in that jurisdiction, whether or not a member of the jurisdiction's bar. 18 Michigan Compiled Laws 600.916. 19 The ABA Standing Committee on Client Protection conducted a survey of unlicensed practice of law programs in 2012, and published its results at http://www.directlaw.com/clientprotectionsurveyupl.pdf. This survey provides information about how UPL rules are enforced.20 GILLERS, note 1,supra, at 992-999.

7

200762123.7 09999/09998-0108

Clients do not see their lawyer of choice as confined to the law of one state or to presence in one state.

By engaging in “virtual practice,” a lawyer can be anywhere and everywhere. Specialization in transactional practice creates both a super-competency and a limitation

on practice by choice. Knowledge of a particular state’s law may be less significant than knowledge of federal law or familiarity with a particular area of business or law, or a client or legal process.

Choice of law rules allow for the ability to contract about applicable law in most commercial transactions. Forum selection clauses are valid in most cases.21

Activities once presumed to be the regulated practice of law are recognized not to be, and may be provided by non-lawyers (including lawyers not licensed to practice law in a jurisdiction.

II. Going Outside the Borders – Multijurisdictional Practice (“MJP”)

In 2000, the ABA Commission on Multijurisdictional Practice (“MJP Commission”) was established predicated on the need to recognize “dynamic change and evolution in nature and scope of legal practice”22 in light of increasing uncertainty when a lawyer’s work could implicate UPL law of a jurisdiction where the lawyer was not licensed. The MJP Commission was motivated by a need to recognize that client needs and legal practices had evolved to the point that the wisdom of UPL rules as applied to licensed lawyers was questionable, and that lawyer regulation needed to respond effectively to that evolution.

The MJP Report noted that lawyers were concerned that literal application of UPL rules would “impede lawyers’ ability to meet their clients’ multi-state and interstate legal needs effectively and sufficiently.”23 The Report further noted that this concern had been sharpened by the decision of the California Supreme Court in Birbrower, Montalbano, Condon & Frank, P.C. v, Superior Court, 949 P. 2d 1 (1998)(“Birbrower”). The Court in Birbrower denied the plaintiff New York law firm recovery of fees for work performed for a California company. The work related to an impending arbitration proceeding in California, which was never convened. The lawyers traveled to California and visited with the client on three occasions, each for several days. Most work of the lawyers was done in their New York offices, however, and there were frequent communications by email, fax, and phone between the New York office and the California client. The court found that the lawyer's work in California was extensive – physically, citing the trips, and virtually, citing the communications; and the facts that the client was a California company, that the arbitration would occur in California, and the fee agreement was governed by California law.

21 The validity of forum selection clauses was addressed in a dispositive manner in Atlantic Marine Construction Co., Inc. v. United States District Court for the Western District of Texas et al., 571 U.S. ____, 134 S. Ct. 568, 187 L. Ed. 2d 487 (2013), a unanimous decision of the United States Supreme Court, in which it was held that the parties’ forum selection clause if contractually valid was to be given full effect by state and federal courts unless a “public-interest” factor exists that would make application of the clause contrary to the public interest, and, in any case, subject to application of the doctrine of forum non conveniens. 22 MJP Report, supra note 3, Introduction.23 Id.

8

200762123.7 09999/09998-0108

Based on recommendation of the MJP Commission, Model Rules 5.5(b), (c), and (d), pertaining to what it termed “multijurisdictional practice,” were added to the Model Rules in 2002.24 The label “Multijurisdictional Practice of Law” was added to the caption of Model Rule 5.5. These Model Rules are focused on practice of law in a jurisdiction by a lawyer not licensed in that jurisdiction, otherwise regarded as UPL. 25

A. Model Rule 5.5(b) is clearly an “inbound” 26 rule, applying to lawyers licensed elsewhere engaged in certain activities in this jurisdiction.

(b) A lawyer who is not admitted to practice in this jurisdiction shall not:(1) except as authorized by these Rules or other law, establish an office or

other systematic and continuous presence in this jurisdiction for the practice of law; or

(2) hold out to the public or otherwise represent that the lawyer is admitted to practice law in this jurisdiction.

This Model Rule provides specific prohibition on certain activity that would ordinarily be considered UPL. It does not itself prohibit UPL or violating other regulation of the profession, as Model Rule 5.5(a) does. However, the Model Rule provides no permissions, and itself does nothing to facilitate multijurisdictional practice. It does not indicate that practicing law without systematic or continuous presence or without representing admission to practice in the jurisdiction is permissible. It is not a limitation on UPL laws themselves. This is a separate rule of professional conduct, effectively prohibiting – as a matter of professional conduct – certain activities in the jurisdiction by a lawyer not admitted in the jurisdiction, for which a lawyer may be disciplined.

In the black letter of this Model Rule, these prohibitions are vague; and this Model Rule depends significantly on the content of Comment for interpretation. The full text of Comment is included along with the full text of Model Rule 5.5 in Appendix A.

Model Rule 5.5(b)(1) forbids a lawyer not licensed here to have a “systematic and continuous presence” here for the practice of law. What qualifies as such a presence?

Clearly, establishing an office is.o What is an “office?”o What constitutes establishing an office?

24 A list of states adopting the so-called multijurisdictional sections of Model Rule 5.5(b), (c), and (d) adopted in 2002 or similar rules as of October 2014 can be found at http://www.americanbar.org/content/dam/aba/administrative/professional_responsibility/quick_guide_5_5.authcheckdam.pdf. As of October 2014, 13 jurisdictions had adopted the Model Rule; 32 have adopted some similar version. Variances among adaptations require careful review of the rule in an applicable jurisdiction. The six jurisdictions not having adopted any form of the MJP rule were: Hawaii, Mississippi, Montana, New York, Texas, and West Virginia. 25 RESTATEMENT §3 contains a simpler statement of the multijurisdictional practice rule, essentially eliminating the constraints of Model Rule 5.5(b) and conditions of 5.5(c)(1), and effectively applying the permissions expressed in Model Rule 5.5(c)(2)-(4) alone. 26 Surely Model Rule 5.5(a) does not apply to the inbound lawyer, for if so, 5.5(b) would be swallowed up, as the activity described in it would be included as unauthorized practice of law.

9

200762123.7 09999/09998-0108

What else is “presence?” Comment [4] says physical presence is not required. Clearly, it can be virtual presence. When Comment [4] was created in 2002, what was understood as non-physical presence? Has technology allowed presence not contemplated then?

What makes presence “systematic?” Is it regular? planned vs. unplanned? purposeful? patterned? recurring?

What is “continuous?” how long? business days plus weekends? weekdays only? every weekend (e.g., an Illinois lawyer goes to summer home in Michigan and works)? something other than “temporary,” like working in a summer vacation cottage for two weeks every year? what interruption is sufficient to avoid “continuous?”

could “temporary” presence be systematic and continuous? See discussion of Model Rule 5.5(c).

Would Model Rule 5.5(b)(2) require a lawyer to disclaim expressly that the lawyer is admitted to practice in this jurisdiction? A look at the definition of practice of law or exceptions to unauthorized practice indicates that disclaimer may be required, and is advisable if not.

What does Model Rule 5.5(b) except as “authorized by these Model Rules or other law?” Model Rule 5.5(b) was packaged with 5.5(c) and (d), which are stated in terms of permissions or authorizations, but only Model Rule 5.5(d) serves as authorization by the Model Rules.

B. Model Rules 5.5(c) and (d) are the multijurisdictional practice rules, providing permissions for lawyers unlicensed in the jurisdiction to provide legal services “here” in certain circumstances. Because the Model Rules of Professional Conduct apply for purposes of lawyer discipline,27 they do not themselves provide any relief from application of other law or rule, such as UPL laws, that would forbid these permitted activities. Those laws must be amended or adapted based on adoption of the Model Rule or similar rule.

(c) A lawyer admitted in another United States jurisdiction, and not disbarred or suspended from practice in any jurisdiction, may provide legal services on a temporary basis in this jurisdiction that:

(1) are undertaken in association with a lawyer who is admitted to practice in this jurisdiction and who actively participates in the matter;

(2) are in or reasonably related to a pending or potential proceeding before a tribunal in this or another jurisdiction, if the lawyer, or a person the lawyer is assisting, is authorized by law or order to appear in such proceeding or reasonably expects to be so authorized;

(3) are in or reasonably related to a pending or potential arbitration, mediation, or other alternative dispute resolution proceeding in this or another jurisdiction, if the services arise out of or are reasonably related to the lawyer’s practice in a jurisdiction in which the lawyer is admitted to practice and are not services for which the forum requires pro hac vice admission; or

(4) are not within paragraphs (c)(2) or (c)(3) and arise out of or are reasonably related to the lawyer’s practice in a jurisdiction in which the lawyer is admitted to practice.

27 Supra Note 7, Scope [19].10

200762123.7 09999/09998-0108

Although the intent of this Model Rule may be understandable, there are some questionable features of it.

The most ambiguous aspect of Model Rule 5.5(c) is what qualifies as “temporary.” No definition is offered. Comment [6] states that there is no single test; and recognizes that what qualifies may depend on the circumstances, such as a lengthy trial or negotiation.

To what is this Model Rule serving as permission? Unless “temporary basis” could nevertheless constitute “systematic and continuous presence” under Model Rule 5.5(b), no Model Rule would itself forbid “temporary” providing of legal services. Unless UPL laws are adapted around such a Model Rule, its function is at best a safe harbor defense in a disciplinary proceeding where adopted.

Model Rule 5.5(c) [as does Model Rule 5.5(d)] permits a lawyer to “provide legal services.” Model Rule 5.5(a) and (b) prohibit “practice of law.” Why were two different presentations of what a lawyer does used, and why have they not been reconciled in subsequent amendments? Do they mean the same thing? How are they different? If either Model Rule 5.5(c) or (d) is an exception to (b), should the two phrases match?

Is Model Rule 5.5(c) intended as an exception to Model Rule 5.5(b) (“as authorized by these Model Rules”) as it seems to be (paraphrased): “A lawyer may have a presence here systematically and continuously on a temporary basis in certain circumstances.” Or is it a stand-alone rule, unrelated to Model Rule 5.5(b)? Would establishing a temporary office qualify as a temporary basis, and thus be permitted under Model Rule 5.5(c) while prohibited under Model Rule 5.5(b)? This Model Rule is not drafted as an exception to Model Rule 5.5(b), and is not to be intended as such. Comment [5] to Model Rule 5.5, in describing the function of Model Rule 5.5(b), says: “With the exception of paragraphs (d)(1) and (d)(2), this Model Rule [the whole of Model Rule 5.5] does not authorize a lawyer to establish an office or other systematic and continuous presence in this jurisdiction without being admitted to practice generally here.”

Model Rule 5.5(c)(1) allows the lawyer to associate an actively participating lawyer licensed in the jurisdiction. This is recognition that retaining active local counsel permits temporarily providing legal services in the jurisdiction, at least as a matter of discipline. Clearly this Model Rule requires more than a phone call for a form or quick advice. Subsections (c)(2) and (3) pertain to litigation or arbitration.

Of greatest interest to transactional lawyers is Model Rule 5.5(c)(4), which allows a lawyer to perform in other jurisdictions legal services relating to or arising from the lawyer's practice in the lawyer's licensing jurisdiction. Comment [14] sets out a variety of factors evidencing such a relationship, providing guidance in understanding this portion of the Model Rule. Summarized, these factors include:

The lawyer's client is previously or currently represented by the lawyer, or is resident or has substantial contacts with the jurisdiction where the lawyer is licensed. The matter although involving other jurisdictions has a significant connection with that jurisdiction.

Significant aspects of the lawyer's work may be conducted in the licensing jurisdiction or a significant aspect of the matter may involve the law of that jurisdiction.

Client activities or legal issues involve other jurisdictions.

11

200762123.7 09999/09998-0108

Services may draw on a lawyer's recognized expertise developed through regular practice of law on behalf of clients in matters involving a particular body of federal, nationally uniform, foreign, or international law. [Note - no mention of state law.]

Model Rule 5.5(d) addresses house counsel or practitioners of certain federal law, and was amended in 2013 to include foreign licensed lawyers:

(d) A lawyer admitted in another United States jurisdiction or in a foreign jurisdiction, and not disbarred or suspended from practice in any jurisdiction or the equivalent thereof, may provide legal services through an office or other systematic and continuous presence in this jurisdiction that :

(1) are provided to the lawyer’s employer or its organizational affiliates; are not services for which the forum requires pro hac vice admission; and, when performed by a foreign lawyer and requires advice on the law of this or another jurisdiction or of the United States, such advice shall be based upon the advice of a lawyer who is duly licensed and authorized by the jurisdiction to provide such advice; or

(2) are services that the lawyer is authorized by federal or other law or rule to provide in this jurisdiction.

This Model Rule provides exceptions to Model Rule 5.5(b) when a lawyer is employed by the client to provide services to it and its organizational affiliates or when authorized by a federal law or rule to practice in jurisdictions in which the lawyer is not admitted. Again, it is to be noted that this is only a rule of professional conduct, and does not resolve statutory or court rule based rules on unauthorized practice providing otherwise, or requirements for registration of house counsel.

The permission in Model Rule 5.5(d)(2) for services authorized by federal or other law has been applied to permit immigration lawyers,28 bankruptcy, patent and trademark and other specialty bar lawyers to practice their specialties in jurisdictions where not admitted to practice otherwise, but these practices must be well circumscribed, and cases demonstrate that it is commonplace to stray into UPL territory as “one thing leads to another.” The application of federal law to a legal matter is not sufficient to qualify for this permission.29

In perspective, these Model Rules represented a difficult first step in recognizing that lawyers do not stay within state lines, and that the occurrence should be intelligently regulated, not prohibited out of hand, to advance legitimate interests of clients and the profession. It is unfortunate that these first steps were housed under the label Unauthorized Practice of Law, thus continuing approbation of practice outside of the lawyer's admission jurisdiction; and that the Model Rules are seen as sanction of certain actions otherwise regarded as unauthorized practice rather than independently constructed principles governing practice in jurisdictions other than that of the lawyer's admission.

28 See e.g., Illinois State Bar Assoc. Opinion 13-08 (October 2013).29 See e.g., In the Matter of Marilyn Sue Scheer 2014 WL 1217969 (Ca Bar, 2014) ruling on the UPL in other states by a CA lawyer providing mortgage modification services under a federal law.

12

200762123.7 09999/09998-0108

C. Adaptations, Variations, and Alternative Rules.

A majority of U.S. jurisdictions have not adopted Model Rule 5.5 as promulgated by the ABA, although at least 45 jurisdictions have adopted a rule based on the Model Rule.30 A few have embraced a more receptive view, while others have been more restrictive than the Model Rule.31 For example, Colorado’s Rule of Professional Conduct 5.5 does not address multijurisdictional practice directly, but refers to a court rule governing admissions to the bar that permits a lawyer who is licensed in another U.S. jurisdiction to practice freely in Colorado on a temporary basis (subject only to pro hac vice requirements) as long as the lawyer does not take up residence in Colorado or establish an office there.32

On the other hand, New Jersey’s Rule 5.5 permits a lawyer not licensed in that state to negotiate the terms of a transaction in furtherance of the lawyer's representation on behalf of an existing client in a jurisdiction in which the lawyer is admitted to practice if the transaction originates in or is otherwise related to a jurisdiction in which the lawyer is admitted to practice.33 The rule also allows practice activity that “arises directly out of the lawyer's representation on behalf of an existing client in a jurisdiction in which the lawyer is admitted to practice, provided that such practice in this jurisdiction is occasional and is undertaken only when the lawyer's disengagement would result in substantial inefficiency, impracticality or detriment to the client.” (NJ RPC 5.5(b)(3)(v)). The rule does not restrict the lawyer to representation of an existing client. Whether the New Jersey rule requires the existing client also to be in the licensed jurisdiction is not altogether clear.

Yet another view is found in Arizona’s version of the rule, which follows the Model Rule but then requires a lawyer engaged in “authorized multijurisdictional practice” to advise the lawyer’s client the lawyer is not admitted in Arizona and to obtain the client’s informed consent to the representation. The term “authorized multijurisdictional practice” is not defined. This could be interpreted to require house counsel or a lawyer practicing federal law, authorized under Arizona’s Rule 5.5(d),as well as lawyers present in the jurisdiction temporarily under Rule 5.5(c), to make the disclaimer and obtain informed consent.

The point of this discussion is that Model Rule 5.5 is a model, only, and rules, if any, on the permissibility of MJP vary from that model in most states; and the variations are often more restrictive than the Model Rule. For this reason, a lawyer whose practice activity involves application of the law of a jurisdiction in which the lawyer is not licensed – or presence in a jurisdiction regardless of what law is being practiced -- needs to be informed about the UPL law and the rules of professional conduct of that jurisdiction. What is permitted “at home” may be a problem elsewhere. And, as will be noted in Section III.B below, a lawyer can be disciplined “at

30 Supra note 23.31 The ABA Center for Professional Responsibility has published a list to some extent indicating variances in state adoptions of Model Rule 5.5 in a chart on state implementation of ABA MJP policies, accessed at http://www.americanbar.org/content/dam/aba/administrative/professional_responsibility/recommendations.authcheckdam.pdf. This is a valuable resource for lawyers engaged in MJP.32 Colorado Court Rule 220 is found at http://www.coloradosupremecourt.com/pdfs/Registration/ Rule%20220.%20Out-of-State%20Attorney%20--%20Conditions%20of%20Practice.pdf . 33 This New Jersey Rule of Professional Conduct 5.5(b)(3)(i) was determined to preclude drafting the document negotiated, in Opinion 49 of the New Jersey Committee on Unauthorized Practice, October 3, 2012, accessed at http://njlaw.rutgers.edu/collections/ethics/cuap/cua49_1.html.

13

200762123.7 09999/09998-0108

home” for violating the RPC elsewhere, even if the conduct elsewhere would not be a violation of the rules “at home.”

The ABA Commission on Ethics 20/20 (the “Ethics 20/20 Commission”) was created in 2009 to fully examine the Model Rules of Professional Conduct and the U.S. system of lawyer regulation in the context of changes in technology and globalization of practice. 34 Efforts to expand Model Rule 5.5 or to create a new rule devoted to U.S. cross-border practice35 were not advanced by the Ethics 20/20 Commission despite extensive consideration of the subject.

III. Risks of Being Outside the Borders

A. Consequences of Unauthorized Practice of Law and Related Conduct.

Consequences of violating UPL rules are often meted out by courts but may also be the subject of disciplinary proceedings.

(1) Under a state statute, UPL may be a crime (see, e.g., Washington Rev Code 2.48.180).

(2) Under a court rule, UPL may be contempt of court (see, e.g., Michigan General Court Rule 16).

(3) UPL is raised as a defense to payment of a fee (see, e.g., Birbrower, Montalbano, Condon & Frank, P.C. v, Superior Court, 949 P. 2d 1 (1998).

(4) Engaging in UPL may bring into question a lawyer’s competence and overall conduct in a malpractice action, and certainly will be a factor in ascertaining competence. The applicable standard of care will likely be that where the lawyer’s conduct occurs or where the conduct has predominant effect. In addition, state long-arm jurisdiction statutes may be asserted in seeking jurisdiction where the effect of the conduct is in question. Compare, Rockwood Select Asset Fund XI (6)-1, LLC v. Devine, Millimet & Branch, No. 13-4112, 2014 BL 125985 (10th Cir. May 6, 2014), and cases cited there, in which the court refused to support long-arm jurisdiction over a lawyer providing a third-party opinion to a recipient in a Montana, with Bill and Sue Beverage v. Pullman & Comley, LLC et al, CV-13-0170-PR (January 24, 2014), and cases cited there, in which the Arizona Supreme Court affirmed the holding of the Court of Appeals, Division One, 232 Ariz. 414, 306 P.3d 71 (2013), that the State of Arizona had jurisdiction over a Connecticut law firm that rendered an opinion to an Arizona resident by reason of the state’s long-arm jurisdiction rules.

34 Information about the work of the Ethics 20/20 Commission can be found at http://www.americanbar.org/groups/professional_responsibility/aba_commission_on_ethics_20_20.html. 35 Various ideas have been advanced to create rules devoted to cross-jurisdiction practice. Some are summarized in Appendix B to this paper, and others are identified in the Issues Paper Concerning Multijurisdictional Practice issued for comment by the Ethics 20/20 Commission, March 29, 2011, accessed at http://www.americanbar.org/content/dam/aba/administrative/ethics_2020/mjp_issues_paper.authcheckdam.pdf. Other approaches include the ABA Model Rule on Admission by Motion and a proposed ABA Model Rule on Practice Pending Admission, neither of which addresses the ebb and flow of a mobile practice or a virtual practice.

14

200762123.7 09999/09998-0108

(5) Engaging in UPL will violate the counterpart to Model Rule 5.5(a) in the lawyer’s licensing jurisdiction and may violate its counterpart to Model Rule 5.5(b) where UPL occurs as well; and also may bring into question violation of other RPC as well (see, e.g., Model Rule 1.1 Competence) and Model Rule 8.4(a).

(6) Model Rule 8.4(a) declares it professional misconduct to violate a Rule of Professional Conduct. Professional misconduct is the basis for lawyer discipline. The Rule violated may be a Rule anywhere that is applicable to the conduct of the lawyer – the scope of reference is not limited to the lawyer’s licensed jurisdiction. Accordingly, actions that violate an applicable rule of another jurisdiction would be sufficient to justify disciplinary action in a jurisdiction where the lawyer is licensed. Note that this rule violation relates to the jurisdiction’s RPC – not to all rules of regulation of the profession. However, because Model Rule 5.5(a) refers to a broader spectrum of rules, violation of that Model Rule brings all lawyer regulation rules into relevance under Model Rule 8.4(a).

(7) Model Rule 8.4(a) also contains a separate statement declaring that assisting another person to violate a Rule of Professional Conduct, or acting through another person to violate such a Model Rule, is misconduct. This portion of the Model Rule should warrant some prior thought when asking a friend for or giving a friend advice when the result could be a violation of Model Rule 5.5.

B. Professional Discipline Beyond Borders.

Professional discipline is the means by which the legal regulatory system addresses lawyer misconduct. Each state has rules of lawyer disciplinary enforcement. Many are modeled to some extent on the ABA Model Rules of Lawyer Disciplinary Enforcement (“MRLDE”) originally adopted by the ABA House of Delegates in 1989, and most recently amended in 2002. Certain changes to the MRLDE were made in 2002 to encompass discipline of lawyers engaged in multijurisdictional practice. Among the grounds for discipline declared by MRLDE are violation or attempted violation of the state’s RPC or any other rule of the jurisdiction regarding professional conduct (MRLDE Rule 9A(1)) and conduct violating applicable rules of professional conduct of another jurisdiction (MRLDE Rule 9A(2)), roughly tracking Model Rules 5.5(b) and 5.5(a), respectively. This disciplinary rule is not limited to RPC, and is broader in scope than Model Rule 8.4(a).

Under MRLDE Rule 6A any lawyer admitted to practice law in “this” jurisdiction and any lawyer not admitted in this jurisdiction who practices law or renders or offers to render any legal services “in this” jurisdiction, is subject to the disciplinary jurisdiction of the supreme court and of the disciplinary board of the jurisdiction. The principle is clear, but the practicality is not. How does this jurisdiction assert disciplinary control over a lawyer not admitted in it? Even if the right to discipline is asserted, how is it effected? The disciplinary process is not like issuing a speeding ticket.

15

200762123.7 09999/09998-0108

Model Rule 8.5 also describes how disciplinary action is implicated by a violation of a RPC.36

(a) Disciplinary Authority. A lawyer admitted to practice in this jurisdiction is subject to the disciplinary authority of this jurisdiction, regardless of where the lawyer's conduct occurs. A lawyer not admitted in this jurisdiction is also subject to the disciplinary authority of this jurisdiction if the lawyer provides or offers to provide any legal services in this jurisdiction. A lawyer may be subject to the disciplinary authority of both this jurisdiction and another jurisdiction for the same conduct.

This is a very simple rule that tracks closely the principles MRLDE 6A, but adds an obvious result: When a lawyer practices in more than one jurisdiction, the lawyer may be disciplined in both the admitting jurisdiction and the jurisdiction where the legal services were provided for the same conduct. This is a rule of multiple jeopardy, although enforcement of discipline of a lawyer not admitted is often not practical. A non-admitted lawyer could not be disbarred or suspended in a jurisdiction where not admitted. However, the admitting jurisdiction is able to take disciplinary action; and, in the case of unauthorized practice of law in another jurisdiction, there is independent basis under Model Rule 5.5(a) to do so.

When the lawyer is disciplined, the rules of multiple jurisdictions can come into play. Model Rule 8.5(b) provides for choice of law:

(b) Choice of Law. In any exercise of the disciplinary authority of this jurisdiction, the rules of professional conduct to be applied shall be as follows:

(1) for conduct in connection with a matter pending before a tribunal, the rules of the jurisdiction in which the tribunal sits, unless the rules of the tribunal provide otherwise; and

(2) for any other conduct, the rules of the jurisdiction in which the lawyer’s conduct occurred, or, if the predominant effect of the conduct is in a different jurisdiction, the rules of that jurisdiction shall be applied to the conduct. A lawyer shall not be subject to discipline if the lawyer’s conduct conforms to the rules of a jurisdiction in which the lawyer reasonably believes the predominant effect of the lawyer’s conduct will occur.

This is a complex rule: the RPC of the place of conduct apply, but if the predominant effect of the conduct is elsewhere, that jurisdiction's RPC apply. Some questions arise:

If the lawyer never physically leaves the admitting jurisdiction but represents a client in a matter in another jurisdiction, is the lawyer's conduct where the lawyer physically is or does it occur where the subject of the representation is? If where the effect of the representation is, can this constitute unauthorized practice – even though the lawyer is never “in” the jurisdiction of effect? Is Model Rule 8.5(b) consistent with Model Rule 5.5(a) or (b)? Has “in” become less geographically based by application of a choice of law mechanic that recognizes effect of conduct, not where the lawyer is when it occurs?

Where do on-line communications occur?

36 The full text of Model Rule 8.5 and Comment is provided in Appendix A.16

200762123.7 09999/09998-0108

Should the lawyer and a sophisticated client be able to agree which rules should apply, as they might be able under the prevailing conflict of laws rules? Would such an agreement present a Model Rule 1.8(a) [business transaction] or 1.8(h) [agreement limiting liability] issue?

This choice of law rule applies only to violation of RPC, not to other rules such as UPL or rules of lawyer disciplinary enforcement (“LDE”), which describe not only a disciplinary system of a jurisdiction but also how disciplinary consequences are to be determined. However, if violation of a rule of lawyer regulation is a violation of a RPC, as it is under Model Rule 8.4(a), then this limitation is effectively meaningless. The Model Rule applies to disciplinary action only in “this” jurisdiction. But if both the admitting and the offended jurisdiction can discipline, the choice of law could be decided differently in each, in theory.

The RPC applied in a disciplinary proceeding in the lawyer's jurisdiction of admission will most likely not be the RPC of that jurisdiction. Comment [6] to Model Rule 8.5 requires the admitting state to apply the RPC of the offended jurisdiction. A lawyer's conduct may comport with the RPC of the lawyer's admitting jurisdiction, but would be subject to discipline nonetheless in the admitting jurisdiction under RPC of the offended jurisdiction. The admitting state's RPC need not be violated if that jurisdiction’s choice-of-law rules call for the offended jurisdiction’s RPC to apply.

Although it seems clearly implicit in Model Rule 8.5 and in MRDLE 6A that the RPC of a jurisdiction where conduct occurs are to apply to a lawyer not admitted there, disciplinary proceedings against a lawyer under RPC of a jurisdiction where the lawyer was not admitted were dismissed by the Ohio Supreme Court in Disciplinary Counsel v. Harris, 137 Ohio St. 3d 1 (2013) for the reason that the out-of-state lawyer had not sworn to abide by the rules of the state where not admitted.37 However, the matter was referred to the UPL disciplinary board for further consideration under the state’s UPL rules.

Model Rule 8.5 does not require the admitting state to take disciplinary action. If the admitting jurisdiction takes disciplinary action, MRLDE Rule 22D says that admitting jurisdiction must impose the same discipline that the disciplining jurisdiction imposed (if lawyer cannot convince the prosecutor that an exception applies) -- even though the admitting jurisdiction's rule of conduct for the offense or sanction for violation may be different, and even though the disciplining jurisdiction has no ability to carry out the sanction imposed. Nothing in MRLDE Rule 22D requires that the lawyer’s conduct in the offense jurisdiction be an offense of the rules of conduct in the admitting jurisdiction; or that discipline for such an offense in the admitting jurisdiction is the same as imposed in the offense jurisdiction.

The Ethics 20/20 Commission identified this choice of law rule for discipline in multijurisdictional application as unclear and arguably problematic. In its Issues Paper, published January 18, 2011,38 the Commission invited comment about change in this Model Rule. Despite a number of suggestions, no change was recommended.

37 Ohio has adopted a Rule substantially similar to Model Rule 8.5(a), but the Court declined to apply the Ohio RPC nevertheless, without analysis of Rule 8.5. 38 Accessed at http://www.americanbar.org/content/dam/aba/migrated/2011_build/professional_responsibility/20111801.authcheckdam.pdf.

17

200762123.7 09999/09998-0108

IV. How Does This Work?

There are numerous cases and ethics opinions to be considered. A sampling of those of that may be demonstrative of the profession’s struggles with changing practice methods and delivery of services is provided here.

In re Carlton, 708 F. Supp. 2d 524 (D. Md. 2010). A member of the DC bar, also admitted to practice in the District Court for the District of Maryland, worked for a DC law firm. She lived in Massachusetts, and did most of her work from home or an office space in Boston, but met with clients in the DC office. She was not a member of the Massachusetts Bar, and did not hold herself out as a Massachusetts lawyer. The MD district court's rule required that a lawyer admitted to its bar must be a member in good standing of the highest court in a state (or District of Columbia) in which the attorney maintains his or her principal law office, or of Maryland. In addressing the apparent failure to satisfy this requirement, it was argued that the lawyer's office was not where she was physically but where she virtually practiced. She received her mail at the DC office and it was forwarded to her in Massachusetts; she used the DC phone number to place and receive calls. She sent and received electronic communications wherever she was. This satisfied the court, which pointed to her nexus to the DC firm as her home for purposes of malpractice coverage, tax obligations, client trust fund obligations; and the data base (files, accounting records, research) and technology on which she relied was in the DC office although she reached them by telecommuting.

By amendment to its court rule 1:21-1(a) in 2013, the New Jersey Supreme Court eliminated a long-standing requirement that a New Jersey admitted lawyer maintain a physical “bona fide” office somewhere in the United States, and approved a virtual office, although requiring some actual location for inspection of files, hand deliveries, and service of process. NJ’s Rule 5.5, however, requires lawyers not licensed in New Jersey to maintain a bona fide office if engaging in multijurisdictional practice in New Jersey. This opinion stopped short of allowing virtual law practice in requiring a physical location for some purposes.

New York City Bar Professional Ethics Committee Formal Opinion 2014-2 approved the use of a street address that is a “virtual office” by a New York licensed lawyer, for purposes of advertising the lawyer’s legal services. The virtual office is real, in that it is a physical location with workspace facilities and equipment and mail handling services; however, the lawyer did not work out of that office except on rare occasion. This opinion, which is purely related to advertising, cautioned that the arrangement could be questioned under professional conduct rules of other jurisdictions. A New York rule interpreted to require that a NY admitted lawyer but residing outside NY must maintain a physical office presence in NY while not having such a requirement for a NY lawyer residing in NY was considered as unconstitutional in Schoenefeld v. New York, 734 F.3d 464 (2d Cir., 2014). The Ethics Opinion noted that it did not by its opinion approve “virtual law practice” that operates primarily over the Internet.

In indicating that virtual law practice – entirely Internet based – was not inconsistent with California Rules of Professional Conduct, the California Bar Standing Committee on

18

200762123.7 09999/09998-0108

Professional Responsibility and Conduct, in Formal Opinion No. 2012-184, noted that virtual practice could raise questions of UPL both for lawyers who are not members of the CA bar and lawyers admitted to the CA bar who could violate UPL rules elsewhere.

Gould v. Florida Bar, 259 F. App'x 208 (11th Cir. 2007). A member of the New York bar opened an office in Florida, and advertised that from that location he wished to counsel persons about New York legal matters only. The court agreed with the Florida Bar that such activity was unlawful under Fla. Stat. §454.23, declaring it unlawful for anyone not licensed to practice law in Florida to practice law within the state. This opinion stands for the proposition that practicing any law is subject to law practice regulation of the jurisdiction in which the “activity” occurs, even if the law practiced was of a jurisdiction where the lawyer was admitted.

In re Mooreland-Rucker, 2010 OK 43, 237 P.3d 784 (OK 2010). A lawyer originally admitted to practice in Oklahoma moved to Texas to be employed in the U.S. Trustee’s office, and was admitted in the federal courts in Texas. After she left her employment, she established a practice in Texas for the practice of bankruptcy law, but was not admitted to the Texas bar. Leaving the practice of law for a while, she allowed her Oklahoma bar membership to lapse. On application for reinstatement, the Supreme Court of Oklahoma held that her practice in Texas violated Oklahoma Rule 5.5(a) and constituted UPL in Oklahoma – even if Texas had not determined that the lawyer’s practice was UPL under Texas rules. This case is a counterpart to Disciplinary Counsel v. Harris, 137 Ohio St. 3d 1 (2013), discussed above, in that here the violation is determined under the law of the admitting jurisdiction for violation of 5.5(a) elsewhere, whereas the Ohio case was considered in the jurisdiction where the lawyer was not admitted.

New Jersey Committee on The Unauthorized Practice of Law, Opinion 49 (October 3, 2012).39 The Committee was asked whether an out-of-state lawyer, representing and out of state buyer, may prepare a contract for the purchase of New Jersey real estate. New Jersey's RPC Rule 5.5 differs from the Model Rule by enumerating specific circumstances under which a lawyer not admitted in New Jersey may practice New Jersey law. One of such circumstances is engaging in negotiation of the terms of a transaction in furtherance of the lawyer's representation of an existing client in a jurisdiction where the lawyer is admitted to practice and the transaction originates or is otherwise related to that jurisdiction. After reviewing precedent that supported the conclusion that preparing a contract for a real estate transaction is the practice of law, the New Jersey committee concluded that the lawyer could negotiate the terms of the contract, but could not prepare it without committing unauthorized practice of law.

In Opinion No 12-09, the Illinois State Bar Association opined that a non-admitted lawyer could not work primarily in Illinois even in an association with an Illinois licensed partner who would act in all Illinois matters and supervise the non-admitted lawyer, who would concentrate his practice in that lawyer’s admitting jurisdiction. The activity under review constituted systematic and continuous presence, in violation of Illinois RPC Rule 5.5(b).

39 Supra, note 32.19

200762123.7 09999/09998-0108

In UPL Advisory Opinion 10-02,40 the Arizona UPL Committee determined that an out-of-state lawyer not admitted in Arizona could not practice law while in Arizona, even if limited to the law of the lawyer’s licensed jurisdiction, other than on the temporary basis allowed under Arizona RPC Rule 5.5.

V. Solutions

A. Know the rules; look at how they are being applied where you practice law AND where you are licensed.

B. Transactional lawyers are not generally “at the table” when rules of professional conduct are drafted and adopted. Transactional lawyers need to be involved in the process if the rules do not work for them.

C. Barriers to lawyer mobility are being attacked by efforts outside the organized bar by entities such as the National Association for the Advancement of Multijurisdictional Practice. Activity of this organization is regularly reported at http://www.mjplaw.org/about_us.html.

40 A copy of this opinion is reproduced in Appendix C, by permission of the State Bar of Arizona.20

200762123.7 09999/09998-0108

APPENDIX A

RULE 5.5 UNAUTHORIZED PRACTICE OF LAW; MULTIJURISDICTIONAL PRACTICE OF LAW

(a) A lawyer shall not practice law in a jurisdiction in violation of the regulation of the legal profession in that jurisdiction, or assist another in doing so.

(b) A lawyer who is not admitted to practice in this jurisdiction shall not:(1) except as authorized by these Rules or other law, establish an office or other systematic and continuous presence in this jurisdiction for the practice of law; or(2) hold out to the public or otherwise represent that the lawyer is admitted to practice law in this jurisdiction.

(c) A lawyer admitted in another United States jurisdiction, and not disbarred or suspended from practice in any jurisdiction, may provide legal services on a temporary basis in this jurisdiction that:

(1) are undertaken in association with a lawyer who is admitted to practice in this jurisdiction and who actively participates in the matter;(2) are in or reasonably related to a pending or potential proceeding before a tribunal in this or another jurisdiction, if the lawyer, or a person the lawyer is assisting, is authorized by law or order to appear in such proceeding or reasonably expects to be so authorized;(3) are in or reasonably related to a pending or potential arbitration, mediation, or other alternative dispute resolution proceeding in this or another jurisdiction, if the services arise out of or are reasonably related to the lawyer’s practice in a jurisdiction in which the lawyer is admitted to practice and are not services for which the forum requires pro hac vice admission; or(4) are not within paragraphs (c)(2) or (c)(3) and arise out of or are reasonably related to the lawyer’s practice in a jurisdiction in which the lawyer is admitted to practice.

(d) A lawyer admitted in another United States jurisdiction or in a foreign jurisdiction, and not disbarred or suspended from practice in any jurisdiction or the equivalent thereof, may provide legal services through an office or other systematic and continuous presence in this jurisdiction that :

(1) are provided to the lawyer’s employer or its organizational affiliates; are not services for which the forum requires pro hac vice admission; and, when performed by a foreign lawyer and requires advice on the law of this or another jurisdiction or of the United States, such advice shall be based upon the advice of a lawyer who is duly licensed and authorized by the jurisdiction to provide such advice; or(2) are services that the lawyer is authorized by federal or other law or rule to provide in this jurisdiction.

(e) For purposes of paragraph (d), the foreign lawyer must be a member in good standing of a recognized legal profession in a foreign jurisdiction, the members of which are admitted to practice as lawyers or counselors at law or the equivalent, and are subject to effective regulation and discipline by a duly constituted professional body or a public authority.

Comment[1] A lawyer may practice law only in a jurisdiction in which the lawyer is authorized to

practice. A lawyer may be admitted to practice law in a jurisdiction on a regular basis or may be

Copyright American Bar Association, 2014. Reprinted with Permission.

200762123.7 09999/09998-0108

authorized by court rule or order or by law to practice for a limited purpose or on a restricted basis. Paragraph (a) applies to unauthorized practice of law by a lawyer, whether through the lawyer’s direct action or by the lawyer assisting another person. For example, a lawyer may not assist a person in practicing law in violation of the rules governing professional conduct in that person’s jurisdiction.

[2] The definition of the practice of law is established by law and varies from one jurisdiction to another. Whatever the definition, limiting the practice of law to members of the bar protects the public against rendition of legal services by unqualified persons. This Rule does not prohibit a lawyer from employing the services of paraprofessionals and delegating functions to them, so long as the lawyer supervises the delegated work and retains responsibility for their work. See Rule 5.3.

[3] A lawyer may provide professional advice and instruction to nonlawyers whose employment requires knowledge of the law; for example, claims adjusters, employees of financial or commercial institutions, social workers, accountants and persons employed in government agencies. Lawyers also may assist independent nonlawyers, such as paraprofessionals, who are authorized by the law of a jurisdiction to provide particular law-related services. In addition, a lawyer may counsel nonlawyers who wish to proceed pro se.

[4] Other than as authorized by law or this Rule, a lawyer who is not admitted to practice generally in this jurisdiction violates paragraph (b)(1) if the lawyer establishes an office or other systematic and continuous presence in this jurisdiction for the practice of law. Presence may be systematic and continuous even if the lawyer is not physically present here. Such a lawyer must not hold out to the public or otherwise represent that the lawyer is admitted to practice law in this jurisdiction. See also Rules 7.1(a) and 7.5(b).

[5] There are occasions in which a lawyer admitted to practice in another United States jurisdiction, and not disbarred or suspended from practice in any jurisdiction, may provide legal services on a temporary basis in this jurisdiction under circumstances that do not create an unreasonable risk to the interests of their clients, the public or the courts. Paragraph (c) identifies four such circumstances. The fact that conduct is not so identified does not imply that the conduct is or is not authorized. With the exception of paragraphs (d)(1) and (d)(2), this Rule does not authorize a U.S. or foreign lawyer to establish an office or other systematic and continuous presence in this jurisdiction without being admitted to practice generally here.

[6] There is no single test to determine whether a lawyer’s services are provided on a “temporary basis” in this jurisdiction, and may therefore be permissible under paragraph (c). Services may be “temporary” even though the lawyer provides services in this jurisdiction on a recurring basis, or for an extended period of time, as when the lawyer is representing a client in a single lengthy negotiation or litigation.

[7] Paragraphs (c) and (d) apply to lawyers who are admitted to practice law in any United States jurisdiction, which includes the District of Columbia and any state, territory or commonwealth of the United States. Paragraph (d) also applies to lawyers admitted in a foreign jurisdiction. The word “admitted” in paragraphs (c), (d) and (e) contemplates that the lawyer is authorized to practice in the jurisdiction in which the lawyer is admitted and excludes a lawyer who while technically admitted is not authorized to practice, because, for example, the lawyer is on inactive status.

[8] Paragraph (c)(1) recognizes that the interests of clients and the public are protected if a lawyer admitted only in another jurisdiction associates with a lawyer licensed to practice in this jurisdiction. For this paragraph to apply, however, the lawyer admitted to practice in this jurisdiction must actively participate in and share responsibility for the representation of the client.

[9] Lawyers not admitted to practice generally in a jurisdiction may be authorized by law or order of a tribunal or an administrative agency to appear before the tribunal or agency. This authority may be granted pursuant to formal rules governing admission pro hac vice or pursuant to informal practice of the tribunal or agency. Under paragraph (c)(2), a lawyer does not violate this

200762123.7 09999/09998-0108

Rule when the lawyer appears before a tribunal or agency pursuant to such authority. To the extent that a court rule or other law of this jurisdiction requires a lawyer who is not admitted to practice in this jurisdiction to obtain admission pro hac vice before appearing before a tribunal or administrative agency, this Rule requires the lawyer to obtain that authority.

[10] Paragraph (c)(2) also provides that a lawyer rendering services in this jurisdiction on a temporary basis does not violate this Rule when the lawyer engages in conduct in anticipation of a proceeding or hearing in a jurisdiction in which the lawyer is authorized to practice law or in which the lawyer reasonably expects to be admitted pro hac vice. Examples of such conduct include meetings with the client, interviews of potential witnesses, and the review of documents. Similarly, a lawyer admitted only in another jurisdiction may engage in conduct temporarily in this jurisdiction in connection with pending litigation in another jurisdiction in which the lawyer is or reasonably expects to be authorized to appear, including taking depositions in this jurisdiction.

[11] When a lawyer has been or reasonably expects to be admitted to appear before a court or administrative agency, paragraph (c)(2) also permits conduct by lawyers who are associated with that lawyer in the matter, but who do not expect to appear before the court or administrative agency. For example, subordinate lawyers may conduct research, review documents, and attend meetings with witnesses in support of the lawyer responsible for the litigation.

[12] Paragraph (c)(3) permits a lawyer admitted to practice law in another jurisdiction to perform services on a temporary basis in this jurisdiction if those services are in or reasonably related to a pending or potential arbitration, mediation, or other alternative dispute resolution proceeding in this or another jurisdiction, if the services arise out of or are reasonably related to the lawyer’s practice in a jurisdiction in which the lawyer is admitted to practice. The lawyer, however, must obtain admission pro hac vice in the case of a court-annexed arbitration or mediation or otherwise if court rules or law so require.

[13] Paragraph (c)(4) permits a lawyer admitted in another jurisdiction to provide certain legal services on a temporary basis in this jurisdiction that arise out of or are reasonably related to the lawyer’s practice in a jurisdiction in which the lawyer is admitted but are not within paragraphs (c)(2) or (c)(3). These services include both legal services and services that nonlawyers may perform but that are considered the practice of law when performed by lawyers.

[14] Paragraphs (c)(3) and (c)(4) require that the services arise out of or be reasonably related to the lawyer’s practice in a jurisdiction in which the lawyer is admitted. A variety of factors evidence such a relationship. The lawyer’s client may have been previously represented by the lawyer, or may be resident in or have substantial contacts with the jurisdiction in which the lawyer is admitted. The matter, although involving other jurisdictions, may have a significant connection with that jurisdiction. In other cases, significant aspects of the lawyer’s work might be conducted in that jurisdiction or a significant aspect of the matter may involve the law of that jurisdiction. The necessary relationship might arise when the client’s activities or the legal issues involve multiple jurisdictions, such as when the officers of a multinational corporation survey potential business sites and seek the services of their lawyer in assessing the relative merits of each. In addition, the services may draw on the lawyer’s recognized expertise developed through the regular practice of law on behalf of clients in matters involving a particular body of federal, nationally-uniform, foreign, or international law. Lawyers desiring to provide pro bono legal services on a temporary basis in a jurisdiction that has been affected by a major disaster, but in which they are not otherwise authorized to practice law, as well as lawyers from the affected jurisdiction who seek to practice law temporarily in another jurisdiction, but in which they are not otherwise authorized to practice law, should consult the [Court Rule on Provision of Legal Services Following Determination of Major Disaster].

[15] Paragraph (d) identifies two circumstances in which a lawyer who is admitted to practice in another United States or a foreign jurisdiction, and is not disbarred or suspended from practice in any jurisdiction, or the equivalent thereof, may establish an office or other systematic and

200762123.7 09999/09998-0108

continuous presence in this jurisdiction for the practice of law. Pursuant to paragraph (c) of this Rule, a lawyer admitted in any U.S. jurisdiction may also provide legal services in this jurisdiction on a temporary basis. See also Rule on Temporary Practice by Foreign Lawyers. Except as provided in paragraphs (d)(1) and (d)(2), a lawyer who is admitted to practice law in another United States or foreign jurisdiction and who establishes an office or other systematic or continuous presence in this jurisdiction must become admitted to practice law generally in this jurisdiction.

[16] Paragraph (d)(1) applies to a U.S. or foreign lawyer who is employed by a client to provide legal services to the client or its organizational affiliates, i.e., entities that control, are controlled by, or are under common control with the employer. This paragraph does not authorize the provision of personal legal services to the employer’s officers or employees. The paragraph applies to in-house corporate lawyers, government lawyers and others who are employed to render legal services to the employer. The lawyer’s ability to represent the employer outside the jurisdiction in which the lawyer is licensed generally serves the interests of the employer and does not create an unreasonable risk to the client and others because the employer is well situated to assess the lawyer’s qualifications and the quality of the lawyer’s work. To further decrease any risk to the client, when advising on the domestic law of a United States jurisdiction or on the law of the United States, the foreign lawyer authorized to practice under paragraph (d)(1) of this Rule needs to base that advice on the advice of a lawyer licensed and authorized by the jurisdiction to provide it.

[17] If an employed lawyer establishes an office or other systematic presence in this jurisdiction for the purpose of rendering legal services to the employer, the lawyer may be subject to registration or other requirements, including assessments for client protection funds and mandatory continuing legal education. See Rule for Registration of In-House Counsel.

[18] Paragraph (d)(2) recognizes that a U.S. or foreign lawyer may provide legal services in a jurisdiction in which the lawyer is not licensed when authorized to do so by federal or other law, which includes statute, court rule, executive regulation or judicial precedent. See, e.g., Rule on Practice Pending Admission.

[19] A lawyer who practices law in this jurisdiction pursuant to paragraphs (c) or (d) or otherwise is subject to the disciplinary authority of this jurisdiction. See Rule 8.5(a).

[20] In some circumstances, a lawyer who practices law in this jurisdiction pursuant to paragraphs (c) or (d) may have to inform the client that the lawyer is not licensed to practice law in this jurisdiction. For example, that may be required when the representation occurs primarily in this jurisdiction and requires knowledge of the law of this jurisdiction. See Rule 1.4(b).

[21] Paragraphs (c) and (d) do not authorize communications advertising legal services in this jurisdiction by lawyers who are admitted to practice in other jurisdictions. Whether and how lawyers may communicate the availability of their services in this jurisdiction is governed by Rules 7.1 to 7.5.

200762123.7 09999/09998-0108

RULE 8.5 DISCIPLINARY AUTHORITY; CHOICE OF LAW

(a) Disciplinary Authority. A lawyer admitted to practice in this jurisdiction is subject to the disciplinary authority of this jurisdiction, regardless of where the lawyer's conduct occurs. A lawyer not admitted in this jurisdiction is also subject to the disciplinary authority of this jurisdiction if the lawyer provides or offers to provide any legal services in this jurisdiction. A lawyer may be subject to the disciplinary authority of both this jurisdiction and another jurisdiction for the same conduct.

(b) Choice of Law. In any exercise of the disciplinary authority of this jurisdiction, the rules of professional conduct to be applied shall be as follows:

(1) for conduct in connection with a matter pending before a tribunal, the rules of the jurisdiction in which the tribunal sits, unless the rules of the tribunal provide otherwise; and(2) for any other conduct, the rules of the jurisdiction in which the lawyer’s conduct occurred, or, if the predominant effect of the conduct is in a different jurisdiction, the rules of that jurisdiction shall be applied to the conduct. A lawyer shall not be subject to discipline if the lawyer’s conduct conforms to the rules of a jurisdiction in which the lawyer reasonably believes the predominant effect of the lawyer’s conduct will occur.

CommentDisciplinary Authority

[1] It is longstanding law that the conduct of a lawyer admitted to practice in this jurisdiction is subject to the disciplinary authority of this jurisdiction. Extension of the disciplinary authority of this jurisdiction to other lawyers who provide or offer to provide legal services in this jurisdiction is for the protection of the citizens of this jurisdiction. Reciprocal enforcement of a jurisdiction’s disciplinary findings and sanctions will further advance the purposes of this Rule. See, Rules 6 and 22, ABA Rules for Lawyer Disciplinary Enforcement. A lawyer who is subject to the disciplinary authority of this jurisdiction under Rule 8.5(a) appoints an official to be designated by this Court to receive service of process in this jurisdiction. The fact that the lawyer is subject to the disciplinary authority of this jurisdiction may be a factor in determining whether personal jurisdiction may be asserted over the lawyer for civil matters.

Choice of Law[2] A lawyer may be potentially subject to more than one set of rules of professional conduct

which impose different obligations. The lawyer may be licensed to practice in more than one jurisdiction with differing rules, or may be admitted to practice before a particular court with rules that differ from those of the jurisdiction or jurisdictions in which the lawyer is licensed to practice. Additionally, the lawyer’s conduct may involve significant contacts with more than one jurisdiction.

[3] Paragraph (b) seeks to resolve such potential conflicts. Its premise is that minimizing conflicts between rules, as well as uncertainty about which rules are applicable, is in the best interest of both clients and the profession (as well as the bodies having authority to regulate the profession). Accordingly, it takes the approach of (i) providing that any particular conduct of a lawyer shall be subject to only one set of rules of professional conduct, (ii) making the determination of which set of rules applies to particular conduct as straightforward as possible, consistent with recognition of appropriate regulatory interests of relevant jurisdictions, and (iii) providing protection from discipline for lawyers who act reasonably in the face of uncertainty.

Copy right American Bar Association 2014. Reprinted with Permission.

200762123.7 09999/09998-0108

[4] Paragraph (b)(1) provides that as to a lawyer's conduct relating to a proceeding pending before a tribunal, the lawyer shall be subject only to the rules of the jurisdiction in which the tribunal sits unless the rules of the tribunal, including its choice of law rule, provide otherwise. As to all other conduct, including conduct in anticipation of a proceeding not yet pending before a tribunal, paragraph (b)(2) provides that a lawyer shall be subject to the rules of the jurisdiction in which the lawyer’s conduct occurred, or, if the predominant effect of the conduct is in another jurisdiction, the rules of that jurisdiction shall be applied to the conduct. In the case of conduct in anticipation of a proceeding that is likely to be before a tribunal, the predominant effect of such conduct could be where the conduct occurred, where the tribunal sits or in another jurisdiction.

[5] When a lawyer’s conduct involves significant contacts with more than one jurisdiction, it may not be clear whether the predominant effect of the lawyer’s conduct will occur in a jurisdiction other than the one in which the conduct occurred. So long as the lawyer’s conduct conforms to the rules of a jurisdiction in which the lawyer reasonably believes the predominant effect will occur, the lawyer shall not be subject to discipline under this Rule. With respect to conflicts of interest, in determining a lawyer’s reasonable belief under paragraph (b)(2), a written agreement between the lawyer and client that reasonably specifies a particular jurisdiction as within the scope of that paragraph may be considered if the agreement was obtained with the client’s informed consent confirmed in the agreement.

[6] If two admitting jurisdictions were to proceed against a lawyer for the same conduct, they should, applying this rule, identify the same governing ethics rules. They should take all appropriate steps to see that they do apply the same rule to the same conduct, and in all events should avoid proceeding against a lawyer on the basis of two inconsistent rules.

[7] The choice of law provision applies to lawyers engaged in transnational practice, unless international law, treaties or other agreements between competent regulatory authorities in the affected jurisdictions provide otherwise.

200762123.7 09999/09998-0108

APPENDIX B

Among the Models for Change

Minkoff, Ronald C., One License for Life: A Paradigm for Multijurisdictional Practice, The Professional Lawyer, vol. 11 no. 3 (Spring 2000).

o Once a driver's license is obtained, driver may keep license for life as long as she does not change primary residence, fulfills pro forma renewal requirements, and does not commit serious or repeated violations.

o License allows driver to drive anywhere in the country – notwithstanding that traffic laws and conditions differ from her home state.

o Driver's licensure is clearly designed to protect the public, and all rationale for preventing UPL is similarly justified.

o Key elements: no change of primary residence; keep in good standing where licensed and where operating; may include foreign drivers.

Association of Corporate Counsel, The Guiding Principle – Freedom of Movement for All Lawyers Across State Boundaries, Response submitted to Ethics 20/20 Commission Working Group – Inbound Foreign Lawyers, July 2010.

o Any lawyer should be free to practice across jurisdictional boundaries on behalf of a client and in the course of the lawyer's practice, and to relocate and apply for simple recognition in a new jurisdiction based on admission and good standing in the lawyer's home and other practice jurisdictions.

o Any individual admitted as a lawyer in a jurisdiction that appropriately regulates the legal profession is a lawyer for this purpose.

o There must be a uniform set of regulations to facilitate practice of law across jurisdictional lines – perhaps overseen by a single clearinghouse/agency charged with determining who is authorized to practice across jurisdictional lines.

o Key elements: may relocate “residence,” simple recognition of presence/residence change; includes foreign lawyers.

National Mobility Agreement (Federation of Law Societies of Canada) – facilitates temporary and permanent mobility of lawyers between all Canada jurisdictions except three northern territories and Quebec.

o Temporary Mobility concepts (“NMA”): lawyers are entitled to practice in a signatory jurisdiction that has

implemented the NMA. must have liability and defalcation coverage. be in good standing in home jurisdiction. may practice 100 days in a calendar year in not-admitted-in jurisdiction. without a permit. do not need to advise host law society that they are providing legal services

on a temporary basis or with respect to the law of that jurisdiction, but “should” communicate with the law society.

lawyers not eligible for mobility without a permit may apply for one. a lawyer who establishes “economic nexus” with a jurisdiction becomes

ineligible for temporary mobility; may apply for permanent mobility.o “Economic nexus” is established when lawyer does anything inconsistent with

temporary mobility, including providing legal services for more than 100 days in a calendar year. opens an office from which to serve the “public.”

200762123.7 09999/09998-0108

opens and operates a trust account. becomes a resident in the jurisdiction.

o Permanent mobility concepts if a lawyer is entitled to practice in signatory jurisdiction that has

implemented the NMA is of good character and in good standing may transfer permanently, or become admitted to practice without having to write exams must meet other qualifications applicable ordinarily for lawyers to be

entitled to practice in the jurisdiction must certify that lawyer has reviewed and understood reading materials

required by the jurisdiction. Territorial Mobility Agreement (Federation of Law Societies of Canada) – governs

permanent mobility to Yukon, Northwest Territories and Nunavit. Quebec Mobility Agreement (Federation of Law Societies of Canada) – provides for

reciprocal mobility of lawyers between common law provinces and territories and civil law province.

200762123.7 09999/09998-0108

APPENDIX C

UPL Advisory Opinion10-02

(February 2010)

Out of State Attorney Practicing Law in the State of Arizona

This is an Advisory Opinion regarding whether an attorney admitted to practice law in a jurisdiction other than Arizona may practice the law of that other jurisdiction while residing in the state of Arizona.1

Issues:

1. May an out-of-state lawyer, not admitted to practice law Arizona but living in Arizona, practice law limited to the law of jurisdictions in which he is admitted?

2. If so, may such work be performed through an existing law firm in the state in which the lawyer is admitted or if the work may be performed in an Arizona office of record, either the office of the out-of-state lawyer or an admitted Arizona attorney? No.

Facts:

An out-of-state lawyer admitted to practice law in states other than Arizona wishes to relocate to Arizona and to practice law of the states in which he is admitted while physically present in Arizona. The lawyer seeks an advisory opinion regarding whether a lawyer may practice the law of states in which he is admitted while physically present in Arizona if the lawyer clearly discloses the jurisdictional limitations of his practice on his letterhead and business cards, does not solicit or advertise to Arizona residents, and does not practice Arizona law. The lawyer further inquires whether such work may be performed in an Arizona office of record, either that of the lawyer admitted to practice out of state or that of an attorney admitted to practice in Arizona.

Relevant Authority:

Arizona Supreme Court Rule 31:Rule 31. Regulation of the Practice of Law(a) Supreme Court Jurisdiction Over the Practice of Law.1. Jurisdiction. Any person or entity engaged in the practice of law or unauthorized practice of law in this state, as defined by these rules, is subject to this court's jurisdiction.2. Definitions.

A. "Practice of law" means providing legal advice or services to or for another by:(1) preparing any document in any medium intended to affect or secure legal rights for a specific person or entity;(2) preparing or expressing legal opinions;

200762123.7 09999/09998-0108

1 Opinions of the Committee are advisory in nature only and are not binding in any disciplinary or other legal proceedings. © 2010 State Bar of Arizona. Reprinted with permission.

(3) representing another in a judicial, quasi-judicial, or administrative proceeding, or other formal dispute resolution process such as arbitration and mediation;(4) preparing any document through any medium for filing in any court, administrative agency or tribunal for a specific person or entity; or(5) negotiating legal rights or responsibilities for a specific person or entity.

B. "Unauthorized practice of law" includes but is not limited to:(1) engaging in the practice of law by persons or entities not authorized to practice pursuant to paragraphs (b) or (c) or specially admitted to practice pursuant to Rule 33(d); or(2) using the designations "lawyer," "attorney at law," "counselor at law," "law," "law office," "J.D.," "Esq.," or other equivalent words by any person or entity who is not authorized to practice law in this state pursuant to paragraphs (b) or ( c) or specially admitted to practice pursuant to Rule 33(d), the use of which is reasonably likely to induce others to believe that the person or entity is authorized to engage in the practice of law in this state.

C. "Legal assistant/paralegal" means a person qualified by education and training who performs substantive legal work requiring a sufficient knowledge of and expertise in legal concepts and procedures, who is supervised by an active member of the State Bar of Arizona, and for whom an active member of the state bar is responsible, unless otherwise authorized by supreme court rule.D. "Mediator" means an impartial individual who is appointed by a court or government entity or engaged by disputants through written agreement, signed by all disputants, to mediate a dispute.

(b)Authority to Practice. Except as hereinafter provided in section (d), no person shall practice law in this state or represent in any way that he or she may practice law in this state unless the person is an active member of the state bar.

(c)Restrictions on Disbarred Attorneys' and Members' Right to Practice. No member who is currently suspended or on disability inactive status and no former member who has been disbarred shall practice law in this state or represent in any way that he or she may practice law in this state.

(d)Exemptions. Notwithstanding the provisions of section (b), but subject to the limitations of section (c) unless otherwise stated:

27. Nothing in these rules shall affect the ability of lawyers licensed in another jurisdiction to engage in conduct that is permitted under ER 5.5 of the rules of professional conduct.

Rule 42: Arizona Rules of Professional Conduct

ER 5.5. Unauthorized Practice of Law

200762123.7 09999/09998-0108

(a) A lawyer shall not practice law in a jurisdiction in violation of the regulation of the legal profession in that jurisdiction, or assist another in doing so.

(b) A lawyer who is not admitted to practice in this jurisdiction shall not:

(1) except as authorized by these Rules or other law, establish an office or other systematic and continuous presence in this jurisdiction for the practice of law, or

(2) hold out to the public or otherwise represent that the lawyer is admitted to practice law in this jurisdiction.

(c) A lawyer admitted in another United States jurisdiction, and not disbarred or suspended from practice in any jurisdiction, may provide legal services on a temporary basis in this jurisdiction that:

(1) are undertaken in association with a lawyer who is admitted to practice in this jurisdiction and who actively participates in the matter.

(2) are in or reasonably related to a pending or potential proceeding before a tribunal in this or another jurisdiction, if the lawyer, or a person the lawyer is assisting, is authorized by law or order to appear in such proceeding or reasonably expects to be so authorized;

(3) are in or reasonably related to a pending or potential arbitration, mediation, or other alternative dispute resolution proceeding in this or another jurisdiction, if the services arise out of or are reasonably related to the lawyer's practice in a jurisdiction in which the lawyer is admitted to practice and are not services for which the forum requires pro hac vice admission; or

(4) are not within paragraphs (c)(2) or (c)(3) and arise out of or are reasonably related to the lawyer's practice in a jurisdiction in which the lawyer is admitted to practice.

(d) A lawyer admitted in another United States jurisdiction, or a lawyer admitted in a jurisdiction outside the United States, not disbarred or suspended from practice in any jurisdiction, and registered pursuant to Rule 38(i) of these rules, may provide legal services in this jurisdiction that:

(1) are provided to the lawyer's employer or its organizational affiliates and are not services for which the forum requires pro hac vice admission, or

(2) are services that the lawyer is authorized to provide by federal law or other law of this jurisdiction.

200762123.7 09999/09998-0108

Rule 31(a)(2)(A) defines the "Practice of law" as providing legal advice or services to orfor another...." The rules do not limit the term "practice of law" to Arizona law. ER

(e)Any attorney who engages in the authorized multijurisdictional practice of law in the State of Arizona under this rule must advise the lawyer's client that the lawyer is not admitted to practice in Arizona, and must obtain the client's informed consent to such representation.

(f) Attorneys not admitted to practice in the State of Arizona, who are admitted to practice law in any other jurisdiction in the United States and who appear in any court of record or before any administrative hearing officer in the State of Arizona, must also comply with Rules of the Supreme Court of Arizona governing pro hac vice admission.

(g)Any attorney who engages in the multijurisdictional practice of law in the State of Arizona, whether authorized in accordance with these Rules or not, shall be subject to the Rules of Professional Conduct and the Rules of the Supreme Court regarding attorney discipline in the State of Arizona.

Discussion:

1. May an out-of-state lawyer, not admitted in Arizona but residing in Arizona, practice law limited to the law of his home-licensed jurisdictions? No.

No person shall practice law in this state or represent in any way that he or she may practice law in this state unless the person is an active member of the state bar or that person's conduct comes within the exemptions to Arizona Supreme Court Rule 31 including exemption 27 permitting legal services as set out in Arizona's multi-jurisdictional practice rules at ER 5.5. Until s/he is admitted to practice law in Arizona, an out-of-state lawyer is a non-lawyer in Arizona and as such can only engage in conduct permitted by the exemptions to Rule 31 or the multi-jurisdictional practice rules at ER 5.5. The multijurisdictional practice rule ER 5.5(c) addresses the practice by attorneys who are in Arizona temporarily. The rules at ER 5.5 do not address and cannot be extended to permit the practice of law by an out-of-state lawyer permanently residing in Arizona unless the lawyer is in-house counsel.

Consistent with the Supremacy Clause and preemption doctrine, Rule 31 exemptions and MJP rules at ER 5.5 permit out-of-state lawyers to practice federal law as authorized by federal law and rules. The Supremacy Clause does not extend to laws of other states. Neither the exceptions to Rule 31 nor the MJP rules at ER 5.5 permit the out-of-state lawyer to engage in the practice of law of the out-of-state lawyer's jurisdiction while s/he resides in Arizona. 5.5(d)(2) assumes the practice of law is not limited to Arizona law. The multi-jurisdictional rule carves out a "safe harbor" in 5.5(d) for practice of law by an out-of-state lawyer stating the "lawyer admitted in another United States jurisdiction ... may provide legal services in this jurisdiction that ... are services that the lawyer is authorized to provide by federal law or other law of this jurisdiction.” The rule permits that lawyer to provide legal services authorized by federal law and the law of Arizona, and does not permit the out-of-state lawyer to provide legal services authorized by another state's jurisdiction. One cannot extend the authorization to engage in the practice of federal law to an authorization to engage in the practice of the out-of-state lawyer's state of admission.

200762123.7 09999/09998-0108

2. If so, may such work be performed through an existing law firm in the state in office of record, either the office of the out-of-state lawyer or an admitted Arizona attorney? No.

An out-of-state lawyer who is not admitted to practice law in Arizona and permanently resides in this state may only engage in the conduct permitted in the exceptions to Rule 31 that do not permit him or her to practice the laws of his or her admitted jurisdiction. Therefore, the out-of-state lawyer may not establish an office in Arizona for the practice of law consistent with ER 5.5(b): "A lawyer who is not admitted to practice in this jurisdiction shall not: (1) except as authorized by these Rules or other law, establish an office or other systematic and continuous presence in this jurisdiction for the practice of law...."

Conclusion:

An out-of-state lawyer, not admitted to practice in Arizona but living in Arizona, may not practice law limited to the law of jurisdictions in which he is licensed. The out-of-state lawyer may not perform the practice of law in an Arizona office of record, either the office of the out-of-state lawyer or an admitted Arizona attorney.

200762123.7 09999/09998-0108